Generalize the inequality $(a_3-a_1)^2+(a_3-a_2)^2+(a_2-a_1)^2\ge b(a_1^2+a_2^2+a_3^2)$, $b>0$ with $a_1+a_2+a_3=0$.

inequalitylinear algebra

To find a $b>0$ that satisfies the inequality of the title, one can use the condition $a_3=-a_1-a_2$, to obtain that $b=3$. I've been trying to find a way to show that

$$(a_n-a_1)^2+(a_n-a_{n-1})^2+(a_{n-1}-a_{n-2})^2+\ldots+(a_3-a_2)^2+(a_2-a_1)^2\ge b(a_1^2+\ldots+a_n^2)$$
for $a_i\in\mathbb{R}$, with $a_1+\ldots+a_n=0$.

For $n=2,3,4$, I can use direct calculations to find a suitable $b$, but for the general case I wasn't able to do much. I was initially trying to find other ways to deal with this inequality. I started with $n=3$, and I rewrote the left hand side of the inequality as

$$\begin{pmatrix}a_1&a_2&a_3\end{pmatrix}^{T}\begin{bmatrix} 2&-1&-1\\-1&2&-1\\-1&-1 &2 \end{bmatrix}\begin{pmatrix}a_1\\a_2\\a_3\end{pmatrix} \tag{1}$$

which however does not help because the eigenvalues of this matrix are (3,3,0). Next, using $a_1+a_2+a_3=0\Rightarrow a_3=-a_1-a_2$, we can essentially reduce the dimension and rewrite (1) as
$$3(a_1+a_2)^2+\begin{pmatrix}a_1&a_2 \end{pmatrix}^{T}\begin{bmatrix} 3&0\\0 &3 \end{bmatrix}\begin{pmatrix}a_1\\a_2\end{pmatrix}\ge b(a_1^2+a_2^2+a_3^2)\tag{2}$$
where $b=3=\lambda_{min}:=$ the minimum eigenvalue of the $2\times2$ matrix above. This approach however, is easy only for small $n$ where you can calculate the corresponding matrices. I wasn't able to find a suitable expression for the general case, and even in that case I would have to show that the resulting matrix has positive eigenvalues.

Any suggestions? Any other solution or approach is more than welcome.

I experimented with the cases $n=4,5$ and noticed that the minimum eigenvalue of (1) (other than the zero) and the minimum eigenvalue of (2) always coincide. Since the matrix that appears in (1) is easy to construct for the $n$-case, is there a way to justify why the minimum eigenvalue (other than zero) can be selected as $b$?

Best Answer

As pointed out by the other answer, when $n\ge3$, the matrix of the quadratic form for the LHS of the inequality is the symmetric circulant matrix $$ C=\pmatrix{2&-1&0&\cdots&0&-1\\ -1&2&-1&0&\ddots&0\\ 0&-1&\ddots&\ddots&\ddots&\vdots\\ \vdots&0&\ddots&\ddots&\ddots&0\\ 0&\ddots&\ddots&\ddots&\ddots&-1\\ -1&0&\cdots&0&-1&2}. $$ Since it is positive semidefinite (because it is diagonally dominant) and $Ce=0$ (where $e$ denotes the vector of ones), if we arrange the eigenvalues of $C$ as $\lambda_1\ge\lambda_2\ge\cdots\ge\lambda_n$, then $\lambda_n=0$ and the maximum possible value of $b$ is $\lambda_{n-1}$, because $(a_1,\ldots,a_n)^T\perp e$.

The eigenvalues of $C$ are $2(1-\Re(\omega^k))$ for $k=0,1,\ldots,n-1$, where $\omega$ is a primitive $n$-th root of unity. Therefore $b=\lambda_{n-1}=2(1-\Re(\omega))=2(1-\cos\frac{2\pi}{n})$.

Related Question